Here are some math olympiad problems I solved in high school, sometimes with remarks and commentary. This is part one, I will publish more problems and solutions over time. There might be a lot of parts… These are mostly solved during 2021-2024, and I will somewhat publish problems in chronological order of when I solved them but not really.
Problem 1, USAMTS 4/1/31
A group of 100 friends stands in a circle. Initially, one person has
2019 mangoes, and no one else has mangoes. The friends split the
mangoes according to the following rules:
sharing: to share, a friend passes two mangoes to the left and one mango
to the right. eating: the mangoes must also be eaten and enjoyed.
However, no friend wants to be selfish and eat too many mangoes. Every
time a person eats a mango, they must also pass another mango to the
right.
A person may only share if they have at least three mangoes, and they
may only eat if they have at least two mangoes. The friends continue
sharing and eating, until so many mangoes have been eaten that no one is
able to share or eat anymore. Show that there are exactly eight people
stuck with mangoes, which can no longer be shared or eaten.
—USAMTS 4/1/31
Solution 1
Index the friends from 0 to 99 clockwise around the circle and let
fm,i denote the amount of mangoes the ith friend has after m
moves of sharing or eating. WLOG f0,0=2019.
Lemma
For any nonnegative integer m, ∑i=0992ifm,i≡2019(mod2100−1).
Proof of Lemma. We proceed by induction. The base case m=0 is
trivial. Consider having friend a∈{0,1,…,99} share or
eat their mangoes on the mth move. If they share, (here in any term
2ifm,i we always take i modulo 100)
We need the modulo 2100−1 to account for boundary conditions when
a∈{0,99}. And thus the lemma is true after move m. □
Now, if no one is able to eat or share after move n, everyone must
have exactly one or zero mangoes.
S=i=0∑992ifn,i≡2019(mod2100−1)
Where fn,i∈{0,1}. Clearly then S=2019. The base 2
repesentation of 2019 contains 8 digit ones. Hence there must be
exactly eight people with mangoes as desired. ■
Remark 1
I remember spending a lot of time on this and was very satisfied with finding the idea.
Problem 2, 1999 Romania TST P4
Let x1,…,xn>0 be positive reals satisfying x1…xn=1. Prove that
n−1+x11+n−1+x21+⋯+n−1+xn1≤1.
—1999 Romania TST Problem 4
Solution 2
n=1 is trivial. Henceforth we assume n≥2. This looks like ∑f(xi) and ideal for Jensen/Karamata, but unfortunately we don’t have ∑xi as a condition, but ∏xi instead.
Let’s force a sum condition by taking logarithms! Write yi=ln(xi) and f(x)=n−1+ex1. Then we wish to prove, subject to ∑yi=0,
∑f(yi)≤1
Of course, we now consider the convexity of f.
f′′(x)=(e2x−(n−1)ex)/(ex+n−1)3.
f has only one inflection point, namely x=ln(n−1). We are in perfect condition to apply the n−1 EV principle. We prove that there exist reals z1=z2=⋯=zn−1, zn also satisfying ∑zi=0 such that ∑f(yi)≤f(zi).
If one or less of yi lie in (ln(n−1),∞), we are done by Jensen’s Inequality on the rest of yi. Then assume for k≥2 and yi nonincreasing, exactly y1,y2,…,yk lie in (ln(n−1),∞), where f is convex. Set tn=y1+y2+⋯+yk−(k−1)ln(n−1) and t1=t2=⋯=tk−1=ln(n−1). Note (tn,t1,t2,…,tk−1)≻(y1,y2,…,yk) and by Karamata’s Inequality,
It remains to prove (abc)3(a+b+c)≤811 Which is
clear by the identities (abc)2≤(3ab+bc+ca)3 and
3abc(a+b+c)≤(ab+bc+ca)2. ■
Remark 3
I remember being really tired but doing this in 10 minutes on my bed somehow. It’s a good problem to test algebraic intuition, maybe kind of easy for A5.
Problem 4, CMO 2018 P3
Two positive integers a and b are prime-related if either a/b or
b/a is prime. Find all positive integers n with at least three
divisors for which it’s possible to arrange all the divisors of n in a
circle, so that any two adjacent divisors are prime-related.
—Canada MO 2018 Problem 3
Solution 4
We claim that the solution set is all positive integers n not a
perfect square or a prime power. Call a valid configuration of n a
circle expansion of n, a tuple (d0,d1,…dm) going
clockwise around the circle where di are the distinct divisors of n
and didi+1modm+1∈{p,p1} for all
i∈{0,1,…,m}.
We first show that n=pa is not circle expandable for some prime p
and positive integer a≥2. Consider 1 on the circle. WLOG p is
adjacent to 1 in the clockwise direction. Then note, p2 must be
adjacent to p, and in general pk is adjacent to pk−1 in the
clockwise direction. However, that means pa, after wrapping around
the circle, is adjacent to 1. Contradiction. We also show that
n=∏i=1kpiai, for distinct primes pi and positive
integers ai, fails when 2∣ai for all ai. Note that there
are an odd amount of divisors on the circle, each
∏i=1kpibi for some distinct tuple of nonnegative
integers (bi)i=1k. Then consider ∑i=1kbi. Starting
at 1, the sum is 0, and increases or decreases by 1 for each clockwise
adjacent divisor. Since there are an odd number of divisors, the divisor
adjacent to 1 counterclockwise must have an even bi sum, impossible
if it is a prime, contradiction.\
\
Now we prove that all n with more than one prime divisor and not a
perfect square works. Then we can write n=pa⋅k, for some
prime p, odd integer a, and positive integer k≥2 such that
p∤k.
Lemma
Any positive integer k≥2 can be weak circle expanded, or have its
divisors be written as a tuple (d0,d1,…dm) where
didi+1modm+1∈{p,p1} for all
i∈{0,1,…,m−1}.
Proof of Lemma. Note we can weakly circle expand qb for prime q
and positive integer b into (1,q,q2,…,qb). Now assume we
can weakly circle expand k and we prove that we can also weakly circle
expand kqb where q∤k, which will finish the proof by the
Fundamental Theorem of Algebra. Consider a weak circle expansion
A=(1,…) of k and let B=(…,1) be A reversed. Note
A+qB+q2A+⋯+qbX is a valid weak circle expansion, where
+ denotes tuple concatenation, multiplication denotes tuple scaling,
and X is A or B when b is even or odd, respectively. □
Then weakly circle expand k into (1,…) and let A=(…)
denote the expansion without the 1 (note the expansion of k contains
two or more divisors), let B be A reversed, and note that
(1)+(p)+(p2)+⋯+(pa)+paA+pa−1B+pa−2A+⋯+p0B
Is a valid circle expansion, where + denotes tuple concatenation, and
multiplication denotes tuple scaling. ■
Problem 5, China MO 2019 P1
For real numbers a,b,c,d,e≥−1 satisfying a+b+c+d+e=5,
determine the minimum and maximum possible values of
S=(a+b)(b+c)(c+d)(d+e)(e+a)
—China MO 2019 Problem 1
Solution 5
The maximum is 288 and the minimum is −512, with equality at
(−1,−1,−1,−1,9) and (−1,−1,−1,4,4) respectively. Let
(x1,x2,x3,x4,x5)=(a+b,c+d,e+a,b+c,d+e). The
conditions become ∑xi=10 and xi+xi+1≤6
(cyclically), where ∏xi=S. We can assume none of xi are
zero. First we prove that
S≤288
We only need to consider the
cases where an even number of xi are negative. If it is zero,
∏xi≤(5∑xi)5=32
If it is two, note that xi≥−2. There must be two cyclically consecutive
positive xi, so that their sum is at most 6 and product thus at most
9. The last xi is clearly at most 8, since
xi+≤6−xi+1≤6−(−2). Hence,
∏xi≤(−2)2⋅9⋅8=288
If four are negative,
the last one is at most 8, and ∏xi≤(−2)4⋅8<288
Now to prove the lower bound, we only consider when one or three of
xi are negative (five is impossible). If it is one, WLOG x1<0,
If it is three, each of the other two xi are at most 8,
∏xi≥(−2)3(8)2=−512
And we are done. ■
Remark 5
I remember someone saying “least contrived chinese inequality:” on the AoPS thread. I cannot agree more.
Problem 6, TSTST 2018 P8
For which positive integers b>2 do there exist infinitely many
positive integers n such that n2 divides bn+1?
—USA Team Selection Test Selection Test Problem 8
Solution 6
The answer is all b such b+1 is not a power of 2. We first prove
that such b work. We prove that if n works, then there exists
odd prime p∤n so that pn works. By Zsigmondy’s theorem, there exists odd prime p
such ordp(b)=2n. Note that p∤n and p∤b. Now,
bpn+1≡(−1)p+1≡0(modp)
And by lifting the
exponent lemma,
vp(bpn+1)=vp(bn+1)+vp(p)≥2
So we
are done (note bpn+1≡0(modn2)). We also easily have at least one such n, simply find an odd prime p such p∣b+1, and note vp(bp+1)=vp(b+1)+1≥2. Now we prove that
b=2t−1 fail. Consider some n>1 that works, and its smallest
prime factor p. Note n must be odd, and p>2 (otherwise
contradiction modulo 4).